LSAT and Law School Admissions Forum

Get expert LSAT preparation and law school admissions advice from PowerScore Test Preparation.

 Administrator
PowerScore Staff
  • PowerScore Staff
  • Posts: 8916
  • Joined: Feb 02, 2011
|
#73116
Complete Question Explanation

Must Be True-FIB. The correct answer choice is (A).

As with most questions that involve filling in a blank in the stimulus, this question requires that we draw a reasonable inference based on the facts presented. In other words, it is a Must Be True question (and since the stem uses qualifying language - "most logically completes the argument" - we can think of this as a softer variant of Must Be True, a Most Strongly Supported question.) Our task is to review the facts presented and draw a conclusion from them.

The premises are that people who don't yet know how their condition will be treated (surgery vs. something else) experience less pain than those who know what's coming, and that the uncertainty in the first group is more stressful than knowing what treatment lies ahead. That second premise is an explicit assumption made by the author. Putting these premises together, we have a correlation between uncertainty, stress, and pain - as uncertainty increases, stress increases, and as stress increases, pain, somewhat surprisingly, decreases. What sort of conclusion might we draw from this evidence? That there is at least some connection, perhaps a causal one, between increased stress and decreased pain. More stress may actually cause a reduction in pain, which seems counter-intuitive.

Answer choice (A): This is the correct answer choice. While the stimulus certainly does not prove that stress causes a reduction in pain, it does suggest that possibility, and that is the inference that we drew. Remember that we can accept something less than total proof, as the question stem fits a Most Strongly Supported model. Since this matches our prephrase, then as surprising as it might sound and as hard as it might be to believe, it must be a contender as you move to the other answer choices.

Answer choice (B): The is no evidence that pain is beneficial, at least not in this stimulus. Stress seems to have a benefit in the form of reduced pain, but we can draw no inferences about what pain may or may not cause.

Answer choice (C): Answer C appeals to our intuition, but is not supported by the facts, which indicate that as least as far as pain levels go, a lack of information might actually have some positive impact on these patients. Also note that this answer is about actively withholding information, while our stimulus is only about patients who don't yet know what lies in store for them, perhaps because their doctors still haven't determined the best course of action.

Answer choice (D): All the patients in the stimulus have reduced blood flow to their hearts, but some have more stress than others, so there is no reason to conclude that stress is either an effect or a cause of their health problems. It appears that stress may be caused not by reduced blood flow, but by knowing how you will be treated for that problem.

Answer choice (E): The stimulus gives us no statistical data to compare the rate of surgery to the incidence of pain experienced by the patients awaiting treatment. No inference can be drawn about which group is more likely to require surgery for their condition.
 ltoulme
  • Posts: 25
  • Joined: Feb 05, 2014
|
#30938
Hi!

For this question, I pre-phrased an answer that connected less pain to already knowing you need surgery. I thought maybe, you're more likely to already know that you need surgery because you're experiencing pain. However, I now see that this pre-phrase doesn't include the premise about stress.

I see how (A) connects the two premises listed in the stimulus (if you don't know what your treatment is, less pain, and more stress), but I don't see how we can make a causal claim from that like (A)?

Is (C) incorrect because it equates pain to the worsening of the heart condition and assumes that the doctors are "withholding information" when really maybe they just haven't decided yet?

Is (E) incorrect because it assumes that the patients still waiting to hear about surgery are not going to need surgery? However, from the stimulus, it's still a possibility that they all will need surgery.

Thanks very much for your help! I just want to be sure I'm on the right track with this analysis!
User avatar
 Jonathan Evans
PowerScore Staff
  • PowerScore Staff
  • Posts: 726
  • Joined: Jun 09, 2016
|
#30976
Hi, Ltoulme,

Great questions! As you observed in your first paragraph, your difficulty with making a prephrase presented some difficulty for you in answering this question. The first step for your success in a future similar situation would be to work on this prephrasing skill. One you recognize that we are dealing with a Must Be True question that asks you essentially to come up with your own conclusion that makes sense in terms of the premises offered ("it would be reasonable to conclude that..."), you need to identify the premises and note any possible connections between the facts provided:
  • P1: Heart condition patients waiting to find out about surgery less likely to experience pain from condition.

    P2: Heart condition patients who already knew about treatment more likely to experience pain.

    P3: Uncertainty more stressful than certainty.
Identify ways you can combine this information together. You know now that the group in P1 has more stress but less pain, whereas the group in P2 has less pain but more stress. You also know from P3 that finding out about the treatment option (or not finding out) is causally connected to the amount of stress. Therefore, what would be a reasonable (not necessarily perfect) conclusion? What do you have a lot of evidence for?

You have evidence sufficient to conclude that it is possible that increased stress may correlate with less pain.

This is the process you should follow (mentally) to arrive at an accurate prephrase. With this prephrase, you have a good match in Answer Choice (A). As you noted we lack any information about the severity of the heart condition that might support Answer Choice (C). Likewise, for Answer Choice (E) we lack any information about the likelihood of requiring surgery.
 medialaw111516
  • Posts: 80
  • Joined: Dec 11, 2018
|
#72053
I cannot figure out for the life of me how A is correct. I didn't love any of the answer, but I'm really struggling with A. I think it's my common sense/real world knowledge that's interfering with me being able to get why A is correct, but I'd still like to know the reason for picking A. I can see why all the other answers are wrong (which technically should be good enough for me to pick A) but I still feel I'm missing something.
 Paul Marsh
PowerScore Staff
  • PowerScore Staff
  • Posts: 290
  • Joined: Oct 15, 2019
|
#72293
Hi Medialaw! This question is asking us to Fill in the Blank, but due to the location of the blank, it's really asking us to come up with a logical conclusion for the argument. Here are essentially the premises that the stimulus presents us: "Uncertainty = less pain. Certainty = more pain. Uncertainty more stressful than certainty." Therefore, what kind of conclusion can we make from those premises? These statements from our stimulus tell us that patients in a more stressful situation (uncertainty about surgery) have less pain than the patients in a less stressful situation (certainty about surgery), so we can combine those premises to say something like, "At least in this instance, stress can sometimes alleviate pain". So that's the sort of conclusion we want to be looking for in our answer choices. Answer Choice (A) says almost exactly that, so that is our right answer. Hope that helps!

Get the most out of your LSAT Prep Plus subscription.

Analyze and track your performance with our Testing and Analytics Package.